Last visit was: 28 Apr 2024, 06:04 It is currently 28 Apr 2024, 06:04

Close
GMAT Club Daily Prep
Thank you for using the timer - this advanced tool can estimate your performance and suggest more practice questions. We have subscribed you to Daily Prep Questions via email.

Customized
for You

we will pick new questions that match your level based on your Timer History

Track
Your Progress

every week, we’ll send you an estimated GMAT score based on your performance

Practice
Pays

we will pick new questions that match your level based on your Timer History
Not interested in getting valuable practice questions and articles delivered to your email? No problem, unsubscribe here.
Close
Request Expert Reply
Confirm Cancel
SORT BY:
Date
avatar
Intern
Intern
Joined: 18 Jun 2011
Posts: 42
Own Kudos [?]: 354 [14]
Given Kudos: 0
Send PM
User avatar
Retired Moderator
Joined: 20 Dec 2010
Posts: 1114
Own Kudos [?]: 4702 [0]
Given Kudos: 376
Send PM
User avatar
Director
Director
Joined: 08 May 2009
Status:There is always something new !!
Affiliations: PMI,QAI Global,eXampleCG
Posts: 552
Own Kudos [?]: 589 [0]
Given Kudos: 10
Send PM
User avatar
Senior Manager
Senior Manager
Joined: 01 Feb 2011
Posts: 309
Own Kudos [?]: 324 [0]
Given Kudos: 42
Send PM
Re: D.S - 700 level question [#permalink]
1. Not sufficient

m = 9 18 27 36...

n =4 2 1...

2. Not sufficient

n = 2 4 6 8 10 12 ...18 ...26

m =18 9 6 ...... respectively

together,

m is multiple of 9 and n is multiple of 2

m 9 18
n 4 2 respectively.
Not sufficient.

Answer is E.
avatar
Intern
Intern
Joined: 24 Jun 2011
Posts: 1
Own Kudos [?]: 1 [1]
Given Kudos: 0
Send PM
Re: D.S - 700 level question [#permalink]
1
Bookmarks
1. m is divisible by 9, hence m can be 9, 18, 36. (1) is not sufficient
2. n is divisible by 9, hence m can be 2, 4, 18. (2) is not sufficient.

m is divisible by 9, and n is divisible by 2: we have two pairs: (9,4) and (18,2). Not sufficient.

Hence E.
User avatar
Manager
Manager
Joined: 20 Jul 2011
Posts: 66
Own Kudos [?]: 208 [1]
Given Kudos: 15
GMAT Date: 10-21-2011
Send PM
Re: D.S - 700 level question [#permalink]
1
Bookmarks
Quote:
The product of positive integers m and n is 36.
What is the value of m + n?

(1) m is evenly divisible by 9.

(2) n is evenly divisible by 2.


m*n=36
m+n=?

Prime factors of 36: 3, 3, 2, 2

From Statement 1
possible values of m: 9, 18 and 36
Insufficient

From Statement 2
posible values of n: 2, 4, 6, 18, 12, 36
Insufficient

Statement 1 + 2
m*n=36
--> m=9, n=4, or
--> m=18, n=2
Insufficient.

Answer: E
User avatar
Manager
Manager
Joined: 13 Apr 2010
Posts: 89
Own Kudos [?]: 56 [0]
Given Kudos: 25
Location: singapore
Concentration: Entrepreneurship,PE,VC
Schools:Wharton,NY Stern,INSEAD,Stanford
 Q42  V10
Send PM
Re: D.S - 700 level question [#permalink]
Plug-in numbers 9*4 , 18*2
+1 for E
avatar
Intern
Intern
Joined: 08 Sep 2011
Posts: 29
Own Kudos [?]: 23 [0]
Given Kudos: 5
Concentration: Finance, Strategy
Send PM
Re: D.S - 700 level question [#permalink]
not sure if this is 700 level, but answer is e.

statement 1: 9, 18 and 36 are all factors of 36 and divisbile by 9, insuff

Statement 2: could be 2, 6, 12, 18, 36 so insuff

Statement 1&2: could be 18 and 2 or 9 and 4. so insuff

i am sure fluke did a better job explaining
Math Revolution GMAT Instructor
Joined: 16 Aug 2015
Posts: 10161
Own Kudos [?]: 16613 [0]
Given Kudos: 4
GMAT 1: 760 Q51 V42
GPA: 3.82
Send PM
Re: The product of positive integers m and n is 36. What is the [#permalink]
Expert Reply
Forget conventional ways of solving math questions. In DS, Variable approach is the easiest and quickest way to find the answer without actually solving the problem.
Remember equal number of variables and independent equations ensures a solution.

The product of positive integers m and n is 36. What is the value of m + n?

(1) m is evenly divisible by 9.
(2) n is evenly divisible by 2.

In the original condition we have 2 variables (m,n) and 1 equation (mn=36) thus we need 1 more equation to match the number ov variables and equations. Since there is 1 each in 1) and 2), there is high probability that D is the answer.

In case of 1), (m,n)=(9,4),(18,2) the answer is not unique and therefore the condition is not sufficient.
In case of 2), ((m,n)=(9,4),(18,2) the answer is not not unique, and therefore the condition is not sufficient

Using both 1) & 2) together, (m,n)=(9,4),(18,2) the answer is not unique, and the condition is not sufficient.
Therefore the answer is E.

Normally for cases where we need 1 more equation, such as original conditions with 1 variable, or 2 variables and 1 equation, or 3 variables and 2 equations, we have 1 equation each in both 1) and 2). Therefore D has a high chance of being the answer, which is why we attempt to solve the question using 1) and 2) separately. Here, there is 59 % chance that D is the answer, while A or B has 38% chance. There is 3% chance that C or E is the answer for the case. Since D is most likely to be the answer according to DS definition, we solve the question assuming D would be our answer hence using 1) and 2) separately. Obviously there may be cases where the answer is A, B, C or E.
User avatar
Intern
Intern
Joined: 21 Mar 2014
Posts: 16
Own Kudos [?]: 14 [1]
Given Kudos: 13
Send PM
Re: The product of positive integers m and n is 36. What is the [#permalink]
1
Kudos
what does evenly divisible means?
if m is evenly divisible by 9, doesn't that mean that m can only be 18 and 36?
Math Expert
Joined: 02 Sep 2009
Posts: 92969
Own Kudos [?]: 619617 [1]
Given Kudos: 81613
Send PM
Re: The product of positive integers m and n is 36. What is the [#permalink]
1
Kudos
Expert Reply
saroshgilani wrote:
what does evenly divisible means?
if m is evenly divisible by 9, doesn't that mean that m can only be 18 and 36?


Evenly divisible simply means divisible, so divisible without a remainder.
User avatar
Intern
Intern
Joined: 21 Mar 2014
Posts: 16
Own Kudos [?]: 14 [0]
Given Kudos: 13
Send PM
Re: The product of positive integers m and n is 36. What is the [#permalink]
Bunuel wrote:
saroshgilani wrote:
what does evenly divisible means?
if m is evenly divisible by 9, doesn't that mean that m can only be 18 and 36?


Evenly divisible simply means divisible, so divisible without a remainder.




thank you Bunuel....
this explains a lot of questions....
Manager
Manager
Joined: 24 Dec 2017
Posts: 142
Own Kudos [?]: 65 [0]
Given Kudos: 48
Location: India
Concentration: Strategy, Real Estate
Schools: Johnson '21
Send PM
Re: The product of positive integers m and n is 36. What is the [#permalink]
M x N = 36, M + N = ?

St1: M/9 = {9,18,27,36}
Insufficient because we don't have any information about N

St2: N/2 = {2,4,6,8,10.....,36}
Same as St1 but here we don't have any information about M

St1 + St2:
Pick values from each statement and check it.
For eg:
9 x 4 = 36, 9 + 4 = 13
18 x 2 = 36, 18 + 2 = 20

Multiple values!!

Hence its E
Director
Director
Joined: 09 Mar 2018
Posts: 783
Own Kudos [?]: 453 [0]
Given Kudos: 123
Location: India
Send PM
Re: The product of positive integers m and n is 36. What is the [#permalink]
guygmat wrote:
The product of positive integers m and n is 36. What is the value of m + n?

(1) m is evenly divisible by 9.
(2) n is evenly divisible by 2.


Take some time

,

Dont Rush



m and n = 36
9 * 4
18 * 2
36 * 1

(1) m is evenly divisible by 9.
18 * 2
36 * 1

2 cases insufficient

(2) n is evenly divisible by 2
9 * 4
18 * 2

2 cases insufficient

Combine
18 * 2
9 * 4

2 cases insufficient

E
User avatar
Non-Human User
Joined: 09 Sep 2013
Posts: 32719
Own Kudos [?]: 822 [0]
Given Kudos: 0
Send PM
Re: The product of positive integers m and n is 36. What is the [#permalink]
Hello from the GMAT Club BumpBot!

Thanks to another GMAT Club member, I have just discovered this valuable topic, yet it had no discussion for over a year. I am now bumping it up - doing my job. I think you may find it valuable (esp those replies with Kudos).

Want to see all other topics I dig out? Follow me (click follow button on profile). You will receive a summary of all topics I bump in your profile area as well as via email.
GMAT Club Bot
Re: The product of positive integers m and n is 36. What is the [#permalink]
Moderator:
Math Expert
92964 posts

Powered by phpBB © phpBB Group | Emoji artwork provided by EmojiOne